16
$\begingroup$

Possible Duplicate:
Number theory proving question?

Dear friends,

Since $b$, $x$, $y$, $z$, $\ldots$ are integers greater than 1, how can we prove that $$ \gcd (b ^ x - 1, b ^ y - 1, b ^ z - 1 ,\ldots)= b ^ {\gcd (x, y, z, .. .)} - 1 $$ ?

Thank you!

Paulo Argolo

  • 0
    This is essentially a duplicate of this previous question coupled with induction http://math.stackexchange.com/questions/7473/number-theory-proving-question2010-11-23
  • 1
    @Timothy: ah, you're right. I should've checked for duplicates.2010-11-23

2 Answers 2